Solution série d'exercices : Ensemble ℚ des nombres rationnels 4e

Classe: 
Quatrième

Exercice 1 Ensemble $\mathbb{Q}$

1) Complétons par $\in\text{ ou }\not\in$

a) $\dfrac{21}{3}=7\in\mathbb{N}\;;\qquad\dfrac{41}{3}\not\in\mathbb{N}$

b) $\dfrac{21}{3}=7.0\in\mathbb{D}\;;\qquad\dfrac{40}{12}\in\mathbb{Q}\;;\qquad\dfrac{125}{374}\in\mathbb{Q}^{+}$

c) $-\dfrac{365}{73}=-5\in\mathbb{Z}\;;\qquad\dfrac{121}{11}\in\mathbb{Q}\;;\qquad\dfrac{42}{6}=7.0\in\mathbb{D}$

d) $15,5\in\mathbb{Q}\;;\qquad\dfrac{41}{3}\not\in\mathbb{D}\;;\qquad\dfrac{3}{4}\in\mathbb{Q}\;;\qquad -\dfrac{45}{3}\not\in\mathbb{N}$

2) Complétons par $\subset\text{ ou }\not\subset$

$\mathbb{N}\subset\mathbb{Q}\;;\qquad\mathbb{Z}\not\subset\mathbb{N}\;;\qquad\mathfrak{D}\subset\mathbb{D}\;;\qquad\mathbb{Q}\not\subset\mathbb{D}$
 

Exercice 2 Le PGCD et le PPMC

1) Calculons $PGCD\;(504\;;\ 492)\ $ et $\ PGCD\;(888\;;\ 777)$
 
Calcul de $PGCD\;(504\;;\ 492)$
 
Décomposons les nombres $504\ $ et $\ 492$ en un produit de facteurs premiers.
 
On a : $504=1\times 2^{3}\times 3^{2}\times 7\quad$ et $\quad 492=1\times 2^{2}\times 3\times 41$
 
Donc,
 
$\begin{array}{rcl}  PGCD\;(504\;;\ 492)&=&1\times 2^{2}\times 3\\\\&=&12\end{array}$
 
D'où, $\boxed{PGCD\;(504\;;\ 492)=12}$
 
Calcul de $PGCD\;(888\;;\ 777)$
 
En décomposant les nombres $888\ $ et $\ 777$ en un produit de facteurs premiers, on obtient :
$$888=1\times 2^{3}\times 3\times 37\quad\text{et}\quad 777=1\times 3\times 7\times 37$$
Par suite,
 
$\begin{array}{rcl}  PGCD\;(888\;;\ 777)&=&1\times 3\times 37\\\\&=&111\end{array}$
 
D'où, $\boxed{PGCD\;(888\;;\ 777)=111}$
 
Simplifions les fractions suivantes : 
 
$A=\dfrac{504}{492}\ $ et $\ B=-\dfrac{888}{777}$
 
Pour simplifier une fraction on utilise le PGCD du numérateur et du dénominateur.
 
Soit $A=\dfrac{504}{492}$
 
Comme $PGCD\;(504\;;\ 492)=12$ alors, $\dfrac{504}{492}=\dfrac{504\div 12}{492\div 12}=\dfrac{42}{41}$
 
Par suite, $\boxed{A=\dfrac{42}{41}}$
 
Soit $B=-\dfrac{888}{777}$
 
Or, $PGCD\;(888\;;\ 777)=111$ donc, $-\dfrac{888}{777}=-\dfrac{888\div 111}{777\div 111}=-\dfrac{8}{7}$
 
D'où, $\boxed{B=-\dfrac{8}{7}}$
 
2) Dans chacun des cas suivants, déterminons :
$$PPCM\;(a\;;\ b)\quad\text{et}\quad PGCD\;(a\;;\ b)$$
1e CAS : $a=504\;;\quad b=492$
 
La décomposition des nombres $504\ $ et $\ 492$ en un produit de facteurs premiers avait donné :
$$504=1\times 2^{3}\times 3^{2}\times 7\quad\text{et}\quad 492=1\times 2^{2}\times 3\times 41$$
Donc,
 
$\begin{array}{rcl}  PPCM\;(504\;;\ 492)&=&1\times 2^{3}\times 3^{2}\times 7\times 41\\\\&=&20664\end{array}$
 
D'où, $\boxed{PPCM\;(504\;;\ 492)=20664}$
 
A la question 1) on avait : $PGCD\;(504\;;\ 492)=12$
 
2e CAS : $a=121\;;\quad b=210$
 
En décomposant les nombres $121\ $ et $\ 210$ en un produit de facteurs premiers, on obtient :
$$121=1\times 11^{2}\quad\text{et}\quad 210=1\times 2\times 3\times 5\times 7$$
Alors,
 
$\begin{array}{rcl}  PPCM\;(121\;;\ 210)&=&1\times 11^{2}\times 2\times 3\times 5\times 7\\\\&=&25410\end{array}$
 
Ainsi, $\boxed{PPCM\;(121\;;\ 210)=25410}$
 
Les nombres $121\ $ et $\ 210$ n'ont qu'un seul facteur premier en commun ; le nombre premier $1.$
 
Donc, $\boxed{PGCD\;(121\;;\ 210)=1}$
 
3) Montrons que $1029$ est un multiple de $147$
 
On a : $1029\div 147=7\ $ et $\ 1029=(147\times 7)+0$
 
Alors, $1029$ est un multiple de $147$ car le reste $r$ est égal à zéro $(r=0)$
 
On en déduit :
 
$\boxed{PGCD\;(1029\;;\ 147)=147}\ $ et $\ \boxed{PPCM\;(1029\;;\ 147)=1029}$
 

Exercice 3 Opération dans $\mathbb{Q}$

1) Calculons les sommes suivantes puis simplifions :
 
On a :
 
$\begin{array}{rcl} A&=&\dfrac{3}{4}+\dfrac{5}{-3}\\ \\&=&\dfrac{3}{4}+\left(-\dfrac{5}{3}\right)\\ \\&=&\dfrac{3}{4}-\dfrac{5}{3}\\ \\&=&\dfrac{3\times 3}{4\times 3}-\dfrac{5\times 4}{3\times 4}\\ \\&=&\dfrac{9}{12}-\dfrac{20}{12}\\ \\&=&\dfrac{9-20}{12}\\ \\&=&\dfrac{-11}{12}\end{array}$
 
D'où, $\boxed{A=-\dfrac{11}{12}}$
 
On a :
 
$\begin{array}{rcl} B&=&\left(\dfrac{-2}{7}\right)+\left(\dfrac{-3}{2}\right)\\ \\&=& \dfrac{-2}{7}-\dfrac{3}{2}\\ \\&=&\dfrac{-2\times 2}{7\times 2}-\dfrac{3\times 7}{2\times 7}\\ \\&=&\dfrac{-4}{14}-\dfrac{21}{14}\\ \\&=& \dfrac{-4-21}{14}\\ \\&=&\dfrac{-25}{14}\end{array}$
 
Par suite, $\boxed{B=-\dfrac{25}{14}}$
 
On a : 
 
$\begin{array}{rcl} C&=&\dfrac{-2}{13}-\dfrac{7}{13}\\ \\&=&\dfrac{-2-7}{13}\\ \\&=&\dfrac{-9}{13}\end{array}$
 
Ainsi, $\boxed{C=-\dfrac{9}{13}}$
 
2) Calculons les différences suivantes puis simplifions
 
On a :
 
$\begin{array}{rcl} A&=&\dfrac{3}{4}-\dfrac{2}{3}\\ \\&=&\dfrac{3\times 3}{4\times 3}-\dfrac{2\times 4}{3\times 4}\\ \\&=&\dfrac{9}{12}-\dfrac{8}{12}\\ \\&=&\dfrac{9-8}{12}\\ \\&=&\dfrac{1}{12}\end{array}$
 
Donc, $\boxed{A=\dfrac{1}{12}}$
 
On a :
 
$\begin{array}{rcl} B&=&3-\left(\dfrac{-3}{2}\right)\\ \\&=&\dfrac{3}{1}+\dfrac{3}{2}\\ \\&=&\dfrac{3\times 2}{1\times 2}+\dfrac{3\times 1}{2\times 1}\\ \\&=&\dfrac{6}{2}+\dfrac{3}{2}\\ \\&=&\dfrac{6+3}{2}\\ \\&=&\dfrac{9}{2}\end{array}$
 
Par suite, $\boxed{B=\dfrac{9}{2}}$
 
On a :
 
$\begin{array}{rcl} C&=&\left(\dfrac{-12}{15}\right)-\left(\dfrac{-7}{15}\right)\\ \\&=&\dfrac{-12}{15}+\dfrac{7}{15}\\ \\&=&\dfrac{-12+7}{15}\\ \\&=&\dfrac{-5}{15}\\ \\&=&\dfrac{-5}{3\times 5}\end{array}$
 
En simplifiant par $5$, on obtient : $\boxed{C=-\dfrac{1}{3}}$
 
3) Calculons les produits suivants (simplifions)
 
a) 
 
$\begin{array}{rcl} A&=&-3\times\dfrac{3}{4}\\ \\&=&\dfrac{-3\times 3}{4}\\ \\&=&\dfrac{-9}{4}\end{array}$
 
D'où, $\boxed{A=-\dfrac{9}{4}}$
 
$\begin{array}{rcl} B&=&3\times \left(\dfrac{-3}{2}\right)\\ \\&=&\dfrac{3\times(-3)}{2}\\ \\&=&\dfrac{-9}{2}\end{array}$
 
Ainsi, $\boxed{B=-\dfrac{9}{2}}$
 
$\begin{array}{rcl} C&=&\left(\dfrac{-2}{15}\right)\times +35\\ \\&=&\dfrac{(-2)\times(+35)}{15}\\ \\&=&\dfrac{-70}{15}\\ \\&=&\dfrac{(-14)\times 5}{3\times 5}\end{array}$
 
Donc, en simplifiant par $5$, on obtient : $\boxed{C=-\dfrac{14}{3}}$
 
b) 
 
$\begin{array}{rcl} A&=&\dfrac{4}{3}\times -\dfrac{9}{12}\\ \\&=&\times\dfrac{-9}{12}\\ \\&=&\dfrac{4\times(-9)}{3\times 12}\\ \\&=&\dfrac{-36}{36}\\ \\&=&-1\end{array}$
 
D'où, $\boxed{A=-1}$
 
$\begin{array}{rcl} B&=&\dfrac{125}{14}\times\dfrac{49}{-50}\\ \\&=&\dfrac{125\times 49}{14\times(-50)}\\ \\&=&\dfrac{(5\times 25)\times(7\times 7)}{(2\times 7)\times(-2\times 25)}\\ \\&=&\dfrac{5\times 7}{2\times(-2)}\\ \\&=&\dfrac{35}{-4}\end{array}$
 
Par suite, $\boxed{B=-\dfrac{35}{4}}$
 
$\begin{array}{rcl} C&=&\dfrac{-248}{4}\times\dfrac{16}{-21}\\ \\&=&\dfrac{-248\times 16}{4\times(-21)}\\ \\&=&\dfrac{-248\times(4\times 4)}{4\times(-21)}\\ \\&=&\dfrac{-248\times 4}{-21}\\ \\&=&\dfrac{-992}{-21}\nonumber\end{array}$
 
D'où, $\boxed{C=\dfrac{992}{21}}$
 
4) Calculons les quotients suivants (simplifions) :
 
a)
 
$\begin{array}{rcl} A&=&\dfrac{-7}{5}\div 3\\ \\&=&\dfrac{-7}{5}\times\dfrac{1}{3}\\ \\&=&\dfrac{-7}{15}\end{array}$
 
Donc, $\boxed{A=-\dfrac{7}{15}}$
 
$\begin{array}{rcl} B&=&\dfrac{4}{6}\div-12\\ \\&=&\dfrac{4}{6}\times\dfrac{1}{-12}\\ \\&=&\dfrac{4}{-72}\\ \\&=&\dfrac{1\times 4}{(-18)\times 4}\end{array}$
 
Ainsi, en simplifiant par 4, on obtient : $\boxed{B=-\dfrac{1}{18}}$
 
$\begin{array}{rcl} C&=&\left(\dfrac{-2}{15}\right)\div -8\\ \\&=&\dfrac{-2}{15}\times\dfrac{1}{-8}\\ \\&=&\dfrac{-2}{-120}\\ \\&=&\dfrac{2}{120}\\ \\&=&\dfrac{1\times 2}{60\times 2}\end{array}$
 
Donc, en simplifiant par $2$, on trouve : $\boxed{C=\dfrac{1}{60}}$
 
b)
 
$\begin{array}{rcl} A&=&\dfrac{\dfrac{2}{3}}{\dfrac{-4}{5}}\\ \\&=&\dfrac{2}{3}\times\dfrac{5}{-4}\\ \\&=&\dfrac{10}{-12}\\ \\&=&\dfrac{2\times 5}{2\times(-6)}\end{array}$
 
Par suite, après simplification par $2$, on obtient : $\boxed{A=-\dfrac{5}{6}}$
 
$\begin{array}{rcl} B&=&\dfrac{\dfrac{5}{7}}{3}\\ \\&=&\dfrac{5}{7}\times\dfrac{1}{3}\\ \\&=&\dfrac{5}{21}\end{array}$
 
D'où, $\boxed{B=\dfrac{5}{21}}$
 
$\begin{array}{rcl} C&=&\dfrac{-5}{\dfrac{7}{-8}}\\ \\&=&-5\times\dfrac{-8}{7}\\ \\&=&\dfrac{40}{7}\end{array}$
 
Ainsi, $\boxed{C=\dfrac{40}{7}}$
 
$\begin{array}{rcl} D&=&-\dfrac{4}{5}\div\dfrac{+14}{25}\\ \\&=&\dfrac{-4}{15}\times\dfrac{25}{14}\\ \\&=&\dfrac{-100}{210}\\ \\&=&\dfrac{(-10)\times 10}{21\times 10}\end{array}$
 
Donc, en simplifiant par $10$, on trouve : $\boxed{D=-\dfrac{10}{21}}$
 
5) Calculons les puissances suivantes (simplifions) :
 
$\begin{array}{rcl} A&=&\left(\dfrac{2}{5}\right)^{5}\\ \\&=&\dfrac{2^{5}}{5^{5}}\\ \\&=&\dfrac{32}{3125}\end{array}$
 
Donc, $\boxed{A=\dfrac{32}{3125}}$
 
$\begin{array}{rcl} B&=&\left(-\dfrac{3}{2}\right)^{3}\times\left(\dfrac{2}{9}\right)^{5}\\ \\&=&\dfrac{(-3)^{3}\times(2)^{5}}{2^{3}\times 9^{5}}\\ \\&=&\dfrac{(-3)^{3}\times 2^{2}}{1\times(3^{2})^{5}}\\ \\&=&\dfrac{(-1)^{3}\times(3)^{3}\times 2^{2}}{(3^{10})}\\ \\&=&\dfrac{(-1)\times 2^{2}}{3^{7}}\\ \\&=&\dfrac{-4}{2187}\end{array}$
 
D'où, $\boxed{B=-\dfrac{4}{2187}}$
 
$\begin{array}{rcl} C&=&\left(+\dfrac{1}{2}\right)^{-5}\\ \\&=&\dfrac{1}{\left(+\dfrac{1}{2}\right)^{5}}\\ \\&=&\dfrac{1}{\dfrac{(1)^{5}}{(2)^{5}}}\\ \\&=&\dfrac{(2)^{5}}{(1)^{5}}\\ \\&=&\dfrac{32}{1}\end{array}$
 
Ainsi, $\boxed{C=32}$

Exercice 4

Dans une classe de $3^{\text{ième}}\;,\ \dfrac{2}{3}$ des élèves désirent poursuivre leurs études en seconde d'enseignement général, $\dfrac{1}{6}$ veulent aller en seconde technologique et les $5$ élèves restant souhaitent aller en seconde professionnelle.
 
1) Calculons la fraction d'élèves qui veulent aller en seconde professionnelle.
 
On sait que : $\dfrac{2}{3}$ des élèves désirent poursuivre leurs études en seconde d'enseignement général. Ce qui équivaut à $\dfrac{4}{6}$ du nombre total d'élèves.
 
De plus, $\dfrac{1}{6}$ des élèves veulent aller en seconde technologique.
 
Soit : $\dfrac{6}{6}$ la proportion totale représentant le nombre total d'élèves.
 
Alors, la fraction d'élèves voulant aller en seconde professionnelle sera donnée par :
 
$\begin{array}{rcl}\dfrac{6}{6}-\left(\dfrac{4}{6}+\dfrac{1}{6}\right)&=&\dfrac{6}{6}-\dfrac{5}{6}\\ \\&=&\dfrac{1}{6}\end{array}$
 
Ainsi, $\dfrac{1}{6}$ des élèves veulent étudier en seconde professionnelle.
 
2) Déterminons le nombre d'élèves de la classe.
 
On sait que : $5$ élèves souhaitent aller en seconde professionnelle. Ce qui correspond à $\dfrac{1}{6}$ du nombre total d'élèves.
 
On peut alors dire que : $\dfrac{1}{6}$ représente 5 élèves.
 
Ainsi, $\dfrac{4}{6}\;,\ \dfrac{1}{6}\ $ et $\ \dfrac{1}{6}$ représenteront le nombre total d'élèves donné par :
 
$\begin{array}{rcl}\dfrac{4}{6}+\dfrac{1}{6}+\dfrac{1}{6}&=&\left(\dfrac{1}{6}+\dfrac{1}{6}+\dfrac{1}{6}+\dfrac{1}{6}\right)+\dfrac{1}{6}+\dfrac{1}{6}\\ \\&=&(5+5+5+5)+5+5\\ \\&=&30\end{array}$
 
Par suite, le nombre d'élèves de la classe est de $30.$
 
3) Déterminons le nombre d'élèves de la classe désirant poursuivre leurs études en seconde d'enseignement général.
 
Comme la classe compte $30$ élèves et que les $\dfrac{2}{3}$ désirent poursuivre leurs études en seconde d'enseignement général donc, on obtient :
 
$\dfrac{2}{3}\times 30=\dfrac{2\times 30}{3}=20$
 
Ce qui signifie que $20$ élèves de la classe désirent poursuivre leurs études en seconde d'enseignement général.

Exercice 5

Déterminons la fraction du rayon de mercure que représente le rayon de la lune.
 
On considère :
 
$R_{\text{terre}}$ : le rayon de la terre
 
$R_{\text{mercure}}$ : le rayon de mercure
 
$R_{\text{lune}}$ : le rayon de la lune
 
On sait que : le rayon de mercure est égal aux $\dfrac{3}{8}$ du rayon de la terre.
 
Ce qui se traduit par :
$$R_{\text{mercure}}=\dfrac{3}{8}R_{\text{terre}}$$
Ainsi, $8R_{\text{mercure}}=3R_{\text{terre}}$
 
Ce qui donne alors :
$$R_{\text{terre}}=\dfrac{8}{3}R_{\text{mercure}}\qquad\text{égalité (*)}$$
Or, le rayon de la lune est égal aux $\dfrac{3}{11}$ du rayon de la terre.
 
Donc,
$$R_{\text{lune}}=\dfrac{3}{11}R_{\text{terre}}$$
En remplaçant $R_{\text{terre}}$ par son expression trouvée dans l'égalité (*), on obtient :
$\begin{array}{rcl} R_{\text{lune}}&=&\dfrac{3}{11}R_{\text{terre}}\\ \\&=&\dfrac{3}{11}\times\dfrac{8}{3}R_{\text{mercure}}\\ \\&=&\dfrac{3\times 8}{11\times 3}R_{\text{mercure}}\\ \\&=&\dfrac{8}{11}R_{\text{mercure}}\end{array}$
 
Par suite, $\boxed{R_{\text{lune}}=\dfrac{8}{11}R_{\text{mercure}}}$
 
Ainsi, le rayon de la lune est égal aux $\dfrac{8}{11}$ du rayon de mercure.

Exercice 6 Problème de la vie courante

Un ordinateur est vendu $12\,600\text{ F}.$ Un tiers de son prix est versé à la commende, un cinquième à la livraison, le reste en dix mensualités identiques.
 
1) Déterminons la fraction du prix de l'ordinateur représentée par le montant d'une mensualité.
 
On appelle $P$ le prix de l'ordinateur, $S_{_{C}}$ la somme versée à la commande, $S_{_{L}}$ la somme versée à la livraison et $S_{_{R}}$ la somme restante, versée en dix mensualités identiques.
 
On a alors :
$$S_{_{R}}=P-(S_{_{C}}+S_{_{L}})$$
Or, on sait que : un tiers du prix est versé à la commande et un cinquième à la livraison. Donc,
$$S_{_{C}}=\dfrac{1}{3}P\quad\text{et}\quad S_{_{L}}=\dfrac{1}{5}P$$
Par suite,
 
$\begin{array}{rcl} S_{_{R}}&=&P-(S_{_{C}}+S_{_{L}})\\ \\&=&p-\left(\dfrac{1}{3}P+\dfrac{1}{5}P\right)\\ \\&=&p-\left(\dfrac{1\times P\times 5}{3\times 5}+\dfrac{1\times P\times 3}{5\times 3}\right)\\ \\&=&p-\dfrac{5P+3P}{15}\\ \\&=&\dfrac{15P}{15}-\dfrac{8P}{15}\\ \\&=&\dfrac{15P-8P}{15}\\ \\&=&\dfrac{7P}{15}\end{array}$
 
D'où, $\boxed{S_{_{R}}=\dfrac{7}{15}P}$
 
Ainsi, le montant restant représente $\dfrac{7}{15}$ du prix de l'ordinateur.
 
Comme le montant d'une mensualité $S_{_{M}}$ est le dixième de la somme restante, soit :
$$S_{_{M}}=\dfrac{S_{_{R}}}{10}$$
alors,
 
$\begin{array}{rcl} S_{_{M}}&=&\dfrac{S_{_{R}}}{10}\\ \\&=&\dfrac{\dfrac{7}{15}P}{10}\\ \\&=&\dfrac{7}{15}P\times\dfrac{1}{10}\\ \\&=&\dfrac{7}{150}P\end{array}$
 
Par conséquent, le montant d'une mensualité représente $\dfrac{7}{150}$ du prix de l'ordinateur. 
 
2) Calculons le montant d'une mensualité.
 
D'après la question 1), le montant $S_{_{M}}$ d'une mensualité est égal à $\dfrac{7}{150}$ du prix de l'ordinateur.
 
Or, l'ordinateur est vendu à $12600\text{ F}$, donc, $S_{_{M}}=\dfrac{7}{150}\times 12600$
 
Soit : $588\text{ F}$
 
Ainsi, le montant de chaque mensualité est de $588\text{ F}.$
 

Exercice 7 : Puissances

Mettons les expressions suivantes sous la forme de Puissances simples.
 
Alors, en appliquant les propriétés sur les puissances, on a :
 
$\begin{array}{rcl} A&=&(2\times 3)^{-4}\times(2^{3})^{-2}\times 3^{2}\times 2^{-2}\\ \\&=& 2^{-4}\times 3^{-4}\times 2^{-6}\times 3^{2}\times 2^{-2}\\ \\&=&2^{-4}\times 2^{-6}\times 2^{-2}\times 3^{-4}\times 3^{2}\\ \\&=& 3^{-4-6-2}\times 3^{-4+2}\\ \\&=&2^{-12}\times 3^{-2}\end{array}$
 
D'où, $\boxed{A=2^{-12}\times 3^{-2}}$
 
$\begin{array}{rcl} B&=&\left(7^{-3}\times 2^{4}\right)^{-2}\times(7^{3})^{-2}\times 21\times 3\\ \\&=& 7^{6}\times 2^{-8}\times 7^{-6}\times 3\times 7\times 3\\ \\&=&2^{-8}\times 7^{6-6+1}\times 3^{2}\\ \\&=&2^{-8}\times 7\times 3^{2}\end{array}$
 
Donc, $\boxed{B=2^{-8}\times 7\times 3^{2}}$
 
$\begin{array}{rcl} C&=&\dfrac{2^{3}\times 3^{-2}\times(2^{-1})^{3}\times 3^{3}}{(3^{2})^{2}\times(2^{2}\times 3)^{+3}}\\ \\&=&\dfrac{2^{3}\times 3^{-2}\times 2^{-3}\times 3^{3}}{3^{4}\times 2^{6}\times 3^{3}}\\ \\&=&\dfrac{2^{3-3}\times 3^{3-2}}{3^{4+3}\times 2^{6}}\\ \\&=&\dfrac{3}{3^{7}\times 2^{6}}\\ \\&=&3\times 3^{-7}\times 2^{-6}\\ \\&=&3^{1-7}\times 2^{-6}\\ \\&=&3^{-6}\times 2^{-6}\\ \\&=&(3\times 2)^{-6}\\ \\&=&6^{-6} \end{array}$
 
Ainsi, $\boxed{C=6^{-6}}$
 
$\begin{array}{rcl} D&=&\dfrac{14\times 3^{-2}\times 0.5\times (2^{-1})^{-3}\times 7^{3}}{(7^{2})^{-2}\times(2^{2}\times 7)^{-3}}\\ \\&=&\dfrac{2\times 7\times 3^{-2}\times\dfrac{1}{2}\times 2^{3}\times 7^{3}}{7^{-4}\times 2^{-6}\times 7^{-3}}\\ \\&=&\dfrac{2^{1+3}\times 7^{1+3}\times 3^{-2}\times 1}{7^{-4-3}\times 2^{-6}\times 2}\\ \\&=&\dfrac{2^{4}\times 7^{4}\times 3^{-2}}{7^{-7}\times 2^{-6+1}}\\ \\&=&\dfrac{2^{4}\times 7^{4}\times 3^{-2}}{7^{-7}\times 2^{-5}}\\ \\&=&2^{4}\times 2^{5}\times 7^{4}\times 7^{7}\times 3^{-2}\\ \\&=&2^{4+5}\times 7^{4+7}\times 3^{-2}\\ \\&=&2^{9}\times 7^{11}\times 3^{-2}\end{array}$
 
D'où, $\boxed{D=2^{9}\times 7^{11}\times 3^{-2}}$

Exercice 8

1) Mettons les expressions suivantes sous la forme de $2^{n}\times 3^{m}\times 5^{p}$, où $n\;,\ m\ $ et $\ p$ sont des entiers.
 
Soit : $C=12\times 36\times 6^{-5}\times 100\times 5^{-3}$
 
Alors, en décomposant les nombres $12\;,\ 36\;,\ 6\ $ et $\ 100$ en un produit de facteurs premiers, on obtient :
 
$12=2^{2}\times 3$
 
$36=2^{2}\times 3^{2}$
 
$6=2\times 3$
 
$100=2^{2}\times 5^{2}$
 
Ainsi, dans l'écriture de $C$, en remplaçant les nombres $12\;,\ 36\;,\ 6\ $ et $\ 100$ par leur expression, on trouve :
 
$\begin{array}{rcl} C&=&12\times 36\times 6^{-5}\times 100\times 5^{-3}\\\\&=&2^{2}\times 3\times 2^{2}\times 3^{2}\times(2\times 3)^{-5}\times 2^{2}\times 5^{2}\times 5^{-3}\\\\&=&2^{2}\times 3^{1}\times 2^{2}\times 3^{2}\times 2^{-5}\times 3^{-5}\times 2^{2}\times 5^{2}\times 5^{-3}\\\\&=&2^{2}\times 2^{2}\times 2^{-5}\times 2^{2}\times 3^{1}\times 3^{2}\times 3^{-5}\times 5^{2}\times 5^{-3}\\\\&=& 2^{2+2-5+2}\times 3^{1+2-5}\times 5^{2-3}\\\\ &=& 2^{1}\times 3^{-2}\times 5^{-1} \end{array}$
 
D'où, $\boxed{C=2^{1}\times 3^{-2}\times 5^{-1}}$
 
Soit : $D=2\times 64\times 6^{-5}\times 100\times 5^{-3}$
 
En décomposant les nombres $64\;,\ 6\ $ et $\ 100$ en un produit de facteurs premiers, on obtient :
 
$64=2^{6}$
 
$6=2\times 3$
 
$100=2^{2}\times 5^{2}$
 
Ainsi, dans l'écriture de $D$, en remplaçant les nombres $64\;,\ 6\ $ et $\ 100$ par leur expression, on obtient :
 
$\begin{array}{rcl} D&=&2\times 64\times 6^{-5}\times 100\times 5^{-3}\\\\&=&2^{1}\times 2^{6}\times(2\times 3)^{-5}\times 2^{2}\times 5^{2}\times 5^{-3}\\\\&=&2^{1}\times 2^{6}\times 2^{-5}\times 3^{-5}\times 2^{2}\times 5^{2}\times 5^{-3}\\\\&=&2^{1}\times 2^{6}\times 2^{-5}\times 2^{2}\times 3^{-5}\times 5^{2}\times 5^{-3}\\\\&=&2^{1+6-5+2}\times 3^{-5}\times 5^{2-3}\\\\&=&2^{4}\times 3^{-5}\times 5^{-1}\end{array}$
 
D'où, $\boxed{D=2^{4}\times 3^{-5}\times 5^{-1}}$
 
2) Donnons une écriture simple de $E\ $ et $\ F.$
 
Soit : $E=\dfrac{a^{2}\times(bc^{3})^{4}}{a^{-2}\times b^{2}\times c^{2}}$ avec ; $a\;,\ b\;,\ c\;,\ n\ $ et $\ m$ différents de zéro.
 
Alors, en appliquant les propriétés sur les puissances, on obtient :
 
$\begin{array}{rcl} E &=& \dfrac{a^{2}(b c^{3})^{4}}{a^{-2}\times b^{2}\times c^{2}}\\\\&=& \dfrac{a^{2}\times b^{4}\times (c^{3})^{4}}{a^{-2}\times b^{2}\times c^{2}}\\\\&=& \dfrac{a^{2}\times b^{4}\times c^{4\times 3}}{a^{-2}\times b^{2}\times c^{2}}\\\\&=& \dfrac{a^{2}\times b^{4}\times c^{12}}{a^{-2}\times b^{2}\times c^{2}}\\\\&=&a^{2}\times b^{4}\times c^{12}\times a^{2}\times b^{-2}\times c^{-2}\\\\&=&a^{2}\times a^{2}\times b^{4}\times b^{-2}\times c^{12}\times c^{-2}\\\\&=&a^{2+2}\times b^{4-2}\times c^{12-2}\\\\&=&a^{4}\times b^{2}\times c^{10}\end{array}$
 
D'où, $\boxed{E=a^{4}\times b^{2}\times c^{10}}$
 
Soit : $F=\dfrac{n^{-3}\times(n\times m)^{3}\times n^{6}}{m^{+5}\times n^{-8}\times m^{-7}}$ avec ; $a\;,\ b\;,\ c\;,\ n\ $ et $\ m$ différents de zéro.
 
Alors, en appliquant les propriétés sur les puissances, on obtient :
 
$\begin{array}{rcl} F&=&\dfrac{n^{-3}\times(n\times m)^{3}\times n^{6}}{m^{+5}\times n^{-8}\times m^{-7}}\\\\&=&\dfrac{n^{-3}\times n^{3}\times m^{3}\times n^{6}}{m^{5}\times n^{-8}\times m^{-7}}\\\\&=&n^{-3}\times n^{3}\times m^{3}\times n^{6}\times m^{-5}\times n^{8}\times m^{7}\\\\&=&n^{-3}\times n^{3}\times n^{6}\times n^{8}\times m^{3}\times  m^{-5}\times m^{7}\\\\&=&n^{-3+3+6+8}\times m^{3-5+7}\\\\&=&n^{14}\times m^{5}\end{array}$
 
D'où, $\boxed{F=m^{5}\times n^{14}}$

Exercice 9

Déterminons le signe de chacun des nombres suivants :
 
$\left(-\dfrac{1}{3}\right)^{4}\;;\qquad\left(-\dfrac{1}{2}\right)^{5}\;;\qquad\left(\dfrac{1}{2}\right)^{-5}\;;\qquad 4^{-8}\;;\qquad -\dfrac{1}{4^{7}}$
 
On rappelle que d'après les propriétés sur les puissances, on a :
 
$-\ $ si $a$ est un nombre rationnel positif et $n$ un entier naturel alors :
 
$\ \ \centerdot\ a^{n}$ est positif.
 
$\ \ \centerdot\ a^{-n}$ est positif.
 
$-\ $ si $a$ est un nombre rationnel négatif et $n$ un entier naturel alors :
 
$\ \ \centerdot\ a^{n}$ est positif si $n$ est un nombre pair ; c'est-à-dire multiple de $2$
 
$\ \ \centerdot\ a^{n}$ est négatif si $n$ est un nombre impair ; c'est-à-dire n'est pas multiple de $2$
 
Ainsi, en appliquant ces propriétés, on obtient :
 
$-\dfrac{1}{3}$ est un nombre rationnel négatif et $4$ est un nombre entier naturel paire.
 
Par conséquent, $\left(-\dfrac{1}{3}\right)^{4}$ est positif.
 
$-\dfrac{1}{2}$ est un nombre rationnel négatif et $5$ est un nombre entier naturel impaire.
 
Donc, $\left(-\dfrac{1}{2}\right)^{5}$ est négatif.
 
On a : $\dfrac{1}{2}$ est un nombre rationnel positif et $5$ est un entier naturel.
 
Alors, $\left(\dfrac{1}{2}\right)^{-5}$ est positif.
 
On a : $4$ est un nombre positif et $8$ un entier naturel.
 
Par conséquent, $4^{-8}$ est un nombre positif.
 
On a : $\dfrac{1}{4^{7}}=\left(\dfrac{1}{4}\right)^{7}$ est un nombre positif.
 
Par conséquent, son opposé $-\dfrac{1}{4^{7}}$ est négatif.

Exercice 10

Mettons les expressions suivantes sous la forme de $a\times 10^{p}$, où $p\in\mathbb{Z}.$
 
On rappelle que si $n\ $ et $\ m$ sont deux entiers relatifs alors, on a :
$$10^{n}\times 10^{m}=10^{n+m}$$
En appliquant cette propriété, on obtient :
 
$\begin{array}{rcl} A&=&10^{7}\times 10^{-4}\times 10^{2}\\\\&=&10^{7-4+2}\\\\&=&10^{5}\end{array}$
 
D'où, $\boxed{A=1\times 10^{5}}$
 
$\begin{array}{rcl} B&=&5.7\times 10^{-7}\times(10^{-5}\times 10^{+2})^{-2}\\\\&=&5.7\times 10^{-7}\times 10^{(-5)\times(-2)}\times 10^{(+2)\times(-2)}\\\\&=&5.7\times 10^{-7}\times 10^{10}\times 10^{-4}\\\\&=&5.7\times 10^{-7+10-4}\\\\&=&5.7\times 10^{-1}\end{array}$
 
D'où, $\boxed{B=5.7\times 10^{-1}}$
 
De plus, on rappelle que si $a\ $ et $\ b$ sont deux nombres et $n$ un entier relatif alors, on a :
$$a\times 10^{n}-b\times 10^{n}=(a-b)\times 10^{n}$$
En appliquant cette propriété, on obtient :
 
$\begin{array}{rcl} C&=&105.7\times 10^{-7}-120\times 10^{-7}\\\\&=&(105.7-120)\times 10^{-7}\\\\&=&-14.3\times 10^{-7}\end{array}$
 
D'où, $\boxed{C=-14.3\times 10^{-7}}$
 
Soit : $D=2.9\times 10^{-1}-17.8\times 10^{-2}$
 
Alors, on peut écrire : $2.9\times 10^{-1}=29\times 10^{-2}.$
 
Ainsi, en remplaçant dans l'expression de $D$, on obtient :
 
$\begin{array}{rcl} D&=&2.9\times 10^{-1}-17.8\times 10^{-2}\\\\&=&29\times 10^{-2}-17.8\times 10^{-2}\\\\&=&(29-17.8)\times 10^{-2}\\\\&=&11.2\times 10^{-2}\end{array}$
 
D'où, $\boxed{D=11.2\times 10^{-2}}$

Exercice 11

Simplifions les expressions suivantes en utilisant les propriétés des puissances de $10.$
 
On a :
 
$\begin{array}{rcl} A&=&\dfrac{10^{-5}\times 10^{2}}{10^{-7}\times 10^{-4}}\\\\&=&10^{-5}\times 10^{2}\times 10^{7}\times 10^{4}\\\\&=&10^{-5+2+7+4}\\\\&=&10^{8}\end{array}$
 
Donc, $\boxed{A=10^{8}}$
 
Soit : $B=\dfrac{8\times 10^{5}\times 25\times 10^{-6}}{20\times(10^{2})^{5}\times 100}.$
 
En réécrivant $20\ $ et $\ 100$ sous forme de puissance de $10$, on obtient : $20=2\times 10^{1}\ $ et $\ 100=10^{2}.$
 
Alors, en remplaçant dans l'expression de $B$, on trouve :
 
$\begin{array}{rcl} B&=&\dfrac{8\times 10^{5}\times 25\times 10^{-6}}{20\times(10^{2})^{5}\times 100}\\\\&=&\dfrac{8\times 10^{5}\times 25\times 10^{-6}}{2\times 10^{1}\times 10^{2\times 5}\times 10^{2}}\\\\&=&\dfrac{8\times 25\times 10^{5}\times 10^{-6}}{2\times 10^{1}\times 10^{10}\times 10^{2}}\\\\&=&\dfrac{200\times 10^{5}\times 10^{-6}\times 10^{-1}\times 10^{-10}\times 10^{-2}}{2}\\\\&=&\dfrac{2\times 100\times 10^{5}\times 10^{-6}\times 10^{-1}\times 10^{-10}\times 10^{-2}}{2}\\\\&=&\dfrac{2\times 10^{2}\times 10^{5}\times 10^{-6}\times 10^{-1}\times 10^{-10}\times 10^{-2}}{2}\\\\&=&\dfrac{2\times 10^{2+5-6-1-10-2}}{2}\\\\&=&\dfrac{2\times 10^{-12}}{2}\\\\&=&\dfrac{2}{2}\times 10^{-12}\\\\&=&10^{-12}\end{array}$
 
D'où, $\boxed{B=10^{-12}}$
 
Soit : $C=\dfrac{0.25+0.5\times 10^{-2}-15\times 10^{-2}}{5\times 10^{-3}}$
 
Alors, on peut écrire : $0.25=25\times 10^{-2}$
 
Ainsi, en remplaçant dans l'expression de $C$, on obtient :
 
$\begin{array}{rcl} C&=&\dfrac{0.25+0.5\times 10^{-2}-15\times 10^{-2}}{5\times 10^{-3}}\\\\&=&\dfrac{25\times 10^{-2}+0.5\times 10^{-2}-15\times 10^{-2}}{5\times 10^{-3}}\\\\&=&\dfrac{(25+0.5-15)\times 10^{-2}}{5\times 10^{-3}}\\\\&=&\dfrac{10.5\times 10^{-2}}{5\times 10^{-3}}\\\\&=&\dfrac{10.5\times 10^{-2}\times 10^{3}}{5}\\\\&=&\dfrac{10.5\times 10^{-2+3}}{5}\\\\&=&\dfrac{10.5\times 10^{1}}{5}\\\\&=&\dfrac{10.5}{5}\times 10^{1}\\\\&=&2.1\times 10^{1}\end{array}$
 
D'où, $\boxed{C=2.1\times 10^{1}}$
 
Soit : $D=\dfrac{4\times 10^{-5}\times 0.5\times 10^{7}}{10^{7}\times 2\times 10^{-9}}$
 
Alors, on a :
 
$\begin{array}{rcl} D&=&\dfrac{4\times 10^{-5}\times 0.5\times 10^{7}}{10^{7}\times 2\times 10^{-9}}\\\\&=&\dfrac{4\times 0.5\times 10^{-5}\times 10^{7}}{2\times 10^{7}\times 10^{-9}}\\\\&=&\dfrac{2\times 10^{-5+7}}{2\times 10^{7-9}}\\\\&=&\dfrac{2\times 10^{2}}{2\times 10^{-2}}\\\\&=&\dfrac{2\times 10^{2}\times 10^{2}}{2}\\\\&=&\dfrac{2\times 10^{2+2}}{2}\\\\&=&\dfrac{2\times 10^{4}}{2}\\\\&=&\dfrac{2}{2}\times 10^{4}\\\\&=&1\times 10^{4}\end{array}$
 
Ainsi, $\boxed{D=10^{4}}$

Exercice 12

Écrivons les expressions suivantes sans le symbole de valeur absolue.
 
On rappelle que :
 
$\ \ |a|=a$ si $a$ est un nombre positif
 
$\ \ |a|=-a$ si $a$ est un nombre négatif
 
Soit : $A=\left|4-\dfrac{9}{7}\right|$
 
Alors, en appliquant cette propriété de la valeur absolue, on obtient :
 
$\begin{array}{rcl} A&=&\left|4-\dfrac{9}{7}\right|\\\\&=&\left|\dfrac{28}{7}-\dfrac{9}{7}\right|\\\\&=&\left|\dfrac{19}{7}\right|\\\\&=&\dfrac{19}{7}\end{array}$
 
D'où, $\boxed{A=\dfrac{19}{7}}$
 
Soit : $B=\left|1-\dfrac{1}{4}\div 7\right|$
 
Alors, en appliquant cette propriété de la valeur absolue, on trouve :
 
$\begin{array}{rcl} B&=&\left|1-\left(\dfrac{1}{4}\div 7\right)\right|\\\\&=&\left|1-\left(\dfrac{1}{4}\times\dfrac{1}{7}\right)\right|\\\\&=&\left|1-\dfrac{1}{28}\right|\\\\&=&\left|\dfrac{28}{28}-\dfrac{1}{28}\right|\\\\&=&\left|\dfrac{28-1}{28}\right|\\\\&=&\left|\dfrac{27}{28}\right|\\\\&=&\dfrac{27}{28}\end{array}$
 
Ainsi, $\boxed{B=\dfrac{27}{28}}$
 
Soit : $C=\left|\dfrac{3}{4}-\dfrac{4}{3}\right|$
 
Alors, en appliquant cette propriété de la valeur absolue, on obtient :
 
$\begin{array}{rcl} C&=&\left|\dfrac{3}{4}-\dfrac{4}{3}\right|\\\\&=&\left|\dfrac{9}{12}-\dfrac{16}{12}\right|\\\\&=&\left|\dfrac{9-16}{12}\right|\\\\&=&\left|\dfrac{-7}{12}\right|\\\\&=&\dfrac{7}{12}\end{array}$
 
D'où, $\boxed{C=\dfrac{7}{12}}$
 
Soit : $D=\left|\dfrac{2}{3}-\dfrac{1}{2}\div 3\right|$
 
Alors, en appliquant cette propriété de la valeur absolue, on trouve :
 
$\begin{array}{rcl} D&=&\left|\dfrac{2}{3}-\left(\dfrac{1}{2}\div 3\right)\right|\\\\&=&\left|\dfrac{2}{3}-\left(\dfrac{1}{2}\times\dfrac{1}{3}\right)\right|\\\\&=&\left|\dfrac{2}{3}-\dfrac{1}{6}\right|\\\\&=&\left|\dfrac{4}{6}-\dfrac{1}{6}\right|\\\\&=&\left|\dfrac{4-1}{6}\right|\\\\&=&\left|\dfrac{3}{6}\right|\\\\&=&\dfrac{3}{6}\\\\&=&\dfrac{1}{2}\end{array}$
 
Donc, $\boxed{D=\dfrac{1}{2}}$

Exercice 13

On considère les nombres rationnels : $a\;,\ b\ $ et $\ c$ tels que : $a>0\;,\ b<0\ $ et $\ c>0.$
 
Écrivons les expressions suivantes sans le symbole de valeur absolue.
 
Comme $a\ $ et $\ c$ sont positifs alors, $|a|=a\ $ et $\ |c|=c.$
 
Comme $b$ est négatif alors, $|b|=-b.$
 
Donc, dans la suite, on va remplacer la valeur absolue de ces nombre par leur valeur.
 
Soit : $A=|a|+|b|-|c|$
 
Ainsi, en remplaçant dans l'expression de $A$, on obtient :
 
$\begin{array}{rcl} A&=&|a|+|b|-|c|\\\\&=&a-b-c\end{array}$
 
D'où, $\boxed{A=a-b-c}$
 
Soit : $B=|-7abc|$
 
On rappelle que si $a\ $ et $\ b$ sont deux nombres rationnels alors :
$$|ab|=|a|\times|b|$$
Donc, en appliquant cette propriété de la valeur absolue, on trouve :
 
$\begin{array}{rcl} B&=&|-7abc|\\\\&=&|-7|\times|a|\times|b|\times|c|\\\\&=&7\times a\times(-b)\times c\\\\&=&-7abc\end{array}$
 
D'où, $\boxed{B=-7abc}$
 
Soit : $C=\left|a\times\dfrac{b}{c}\right|$
 
On rappelle que $b\ $ et $\ c$ sont deux nombres rationnels avec $c\neq 0$ alors :
$$\left|\dfrac{b}{c}\right|=\dfrac{|b|}{|c|}$$
Donc, en appliquant cette propriété de la valeur absolue, on obtient :
 
$\begin{array}{rcl} C&=&\left|a\times\dfrac{b}{c}\right|\\\\&=&|a|\times\left|\dfrac{b}{c}\right|\\\\&=&|a|\times\dfrac{|b|}{|c|}\\\\&=&a\times\dfrac{-b}{c}\\\\&=&\dfrac{-ab}{c}\end{array}$
 
Ainsi, $\boxed{C=-\dfrac{ab}{c}}$
 
Soit : $D=|-a+b|$
 
Or, on sait que $a$ est positif donc, $-a$ est négatif.
 
Comme $b$ est négatif alors, $-a+b$ qui est la somme de deux nombres négatifs est aussi négatif.
 
Par suite, d'après la définition de la valeur absolue, on a :
 
$\begin{array}{rcl} D&=&|-a+b|\\\\&=&-(-a+b)\\\\&=&a-b\end{array}$
 
Ainsi, $\boxed{D=a-b}$

Exercice 14

1) Dans chacun des cas ci-dessous, nous allons voir si $A$ est égale $B$
 
a) $A=\dfrac{5}{6}\ $ et $\ B=\dfrac{30}{36}$
 
Dans l'écriture de $B$ on remarque que le numérateur et le dénominateur sont divisibles par $6.$
 
Donc, en simplifiant par $6$, on obtient :
 
$B=\dfrac{30}{36}=\dfrac{30\div 6}{36\div 6}=\dfrac{5}{6}$
 
D'où, $B=\dfrac{5}{6}$
 
Par conséquent, $\boxed{A=B}$
 
b) $A=\dfrac{-7}{12}\ $ et $\ B=\dfrac{35}{-60}$
 
Soit : $A=\dfrac{-7}{12}$
 
Alors, en multipliant le numérateur et le dénominateur de $A$ par $5$, on obtient :
 
Donc, en simplifiant par $5$, on trouve :
 
$A=\dfrac{-7}{12}=\dfrac{-7\times 5}{12\times 5}=\dfrac{-35}{60}$
 
Ainsi, $A=\dfrac{35}{-60}$
 
Par conséquent, $\boxed{A=B}$
 
2) Comparons les nombres rationnels suivants en utilisant deux méthodes différentes.
 
a) $\dfrac{5}{6}\ $ et $\ -\dfrac{2}{5}$
 
En effet, on sait que tout nombre positif est plus grand que tout nombre négatif.
 
Or, $\dfrac{5}{6}>0\ $ et $\ -\dfrac{2}{5}<0.$
 
Donc, $\dfrac{5}{6}$ est plus grand que $-\dfrac{2}{5}$
 
b) $\dfrac{2}{7}\ $ et $\ \dfrac{3}{8}$
 
Par calcul direct, on a :
 
$\dfrac{2}{7}=0.28\ $ et $\ \dfrac{3}{8}=0.37$
 
Comme $0.37$ est supérieur à $0.28$ alors, $\dfrac{3}{8}$ est plus grand que $\dfrac{2}{7}$
 
Autrement, en réduisant au même dénominateur, on obtient :
 
$\dfrac{2}{7}=\dfrac{2\times 8}{7\times 8}=\dfrac{16}{56}$
 
$\dfrac{3}{8}=\dfrac{3\times 7}{8\times 7}=\dfrac{21}{56}$
 
Or, on sait que si deux fractions ont le même dénominateur, la plus grande est celle qui a le plus grand numérateur.
 
Comme $21>16$ alors, $\dfrac{21}{56}$ est plus grand que $\dfrac{16}{56}.$
 
Par conséquent, $\dfrac{3}{8}$ est plus grand que $\dfrac{2}{7}$
 
c) $5.1\ $ et $\ \dfrac{14}{3}$
 
Par calcul direct, on a : $\dfrac{14}{3}=4.66$
 
Comme $5.1$ est supérieur à $4.66$ alors, $5.1$ est plus grand que $\dfrac{14}{3}$
 
Autrement, on a : $5.1=\dfrac{51}{10}$
 
Donc, en réduisant au même dénominateur, on obtient :
 
$\dfrac{51}{10}=\dfrac{51\times 3}{10\times 3}=\dfrac{153}{30}$
 
$\dfrac{14}{3}=\dfrac{14\times 10}{3\times 10}=\dfrac{140}{30}$
 
Or, on sait que si deux fractions ont le même dénominateur, la plus grande est celle qui a le plus grand numérateur.
 
Comme $153>140$ alors, $\dfrac{153}{30}$ est plus grand que $\dfrac{140}{30}.$
 
Par conséquent, $5.1$ est plus grand que $\dfrac{14}{3}$

Exercice 15

Rangeons les nombres rationnels ci-dessous dans l'ordre croissant :
$$\dfrac{8}{7}\;;\quad\dfrac{5}{8}\;;\quad\dfrac{7}{8}\;;\quad\dfrac{8}{6}\;;\quad\dfrac{8}{5}\ \text{ et }\ \dfrac{6}{8}$$
En effet, on remarque que :
 
$\dfrac{8}{7}\;;\ \dfrac{8}{6}\;;\ \dfrac{8}{5}$ ont même numérateur.
 
Or, si deux fractions ont le même numérateur, la plus grande est celle qui a le plus petit dénominateur.
 
Donc,
$$\dfrac{8}{7}<\dfrac{8}{6}<\dfrac{8}{5}$$
De la même manière, on constate que :
 
$\dfrac{5}{8}\;;\ \dfrac{7}{8}\;;\ \dfrac{6}{8}$ ont même dénominateur
 
Or, si deux fractions ont le même dénominateur, la plus grande est celle qui a le plus grand numérateur.
 
Donc,
$$\dfrac{5}{8}<\dfrac{6}{8}<\dfrac{7}{8}$$
Par ailleurs :
 
$\dfrac{7}{8}<1$ car le numérateur $7$ est inférieur au dénominateur $8.$
 
$\dfrac{8}{7}>1$ car le numérateur $8$ est supérieur au dénominateur $7.$
 
Ainsi, on obtient :
$$\dfrac{5}{8}<\dfrac{6}{8}<\dfrac{7}{8}<1\ \text{ et }\ 1<\dfrac{8}{7}<\dfrac{8}{6}<\dfrac{8}{5}$$
Par suite,
$$\dfrac{5}{8}<\dfrac{6}{8}<\dfrac{7}{8}<1<\dfrac{8}{7}<\dfrac{8}{6}<\dfrac{8}{5}$$
D'où, un rangement de ces nombres rationnels dans l'ordre croissant est donné par :
$$\dfrac{5}{8}<\dfrac{6}{8}<\dfrac{7}{8}<\dfrac{8}{7}<\dfrac{8}{6}<\dfrac{8}{5}$$

Exercice 16

On considère les nombres rationnels suivants :
$$\dfrac{64}{192}\;;\quad\dfrac{18}{84}\;;\quad +\dfrac{84}{28}\;;\quad\dfrac{7}{21}\;;\quad -\dfrac{120}{160}\;;\quad -\dfrac{-16}{-48}\ \text{ et }\ \dfrac{210}{-441}$$
1) Simplifions l'écriture de chacun des nombres rationnels ci-dessus.
 
En effet, on sait que pour simplifier une fraction, on peut utiliser le $PGCD$ du numérateur et du dénominateur.
 
Soit à simplifier $\dfrac{64}{192}$
 
En décomposant les nombres $64\ $ et $\ 192$ en un produit de facteurs premiers, on obtient : $64=1\times 2^{6}\quad$ et $\quad 192=1\times 2^{6}\times 3$
 
Par suite,
 
$\begin{array}{rcl}  PGCD\;(64\;;\ 192)&=&1\times 2^{6}\\\\&=&64\end{array}$
 
Donc, $\boxed{PGCD\;(64\;;\ 192)=64}$
 
Comme $PGCD\;(64\;;\ 192)=64$ alors, en divisant le numérateur et le dénominateur par $64$, on obtient :
 
$\dfrac{64}{192}=\dfrac{64\div 64}{192\div 64}=\dfrac{1}{3}$
 
Ainsi, $\boxed{\dfrac{64}{192}=\dfrac{1}{3}}$
 
Soit à simplifier $\dfrac{18}{84}$
 
En décomposant les nombres $18\ $ et $\ 84$ en un produit de facteurs premiers, on obtient : $18=1\times 2\times 3^{2} \quad$ et $\quad 84=1\times 2^{2}\times 3\times 7$
 
Ainsi,
 
$\begin{array}{rcl}  PGCD\;(18\;;\ 84)&=&1\times 2\times 3\\\\&=&6\end{array}$
 
Donc, $\boxed{PGCD\;(18\;;\ 84)=6}$
 
Comme $PGCD\;(18\;;\ 84)=6$ alors, en divisant le numérateur et le dénominateur par $6$, on trouve :
 
$\dfrac{18}{84}=\dfrac{18\div 6}{84\div 6}=\dfrac{3}{14}$
 
D'où, $\boxed{\dfrac{18}{84}=\dfrac{3}{14}}$
 
Soit à simplifier $+\dfrac{84}{28}$
 
En décomposant les nombres $28\ $ et $\ 84$ en un produit de facteurs premiers, on obtient : $28=1\times 2^{2}\times 7\quad$ et $\quad 84=1\times 2^{2}\times 3\times 7$
 
Alors,
 
$\begin{array}{rcl}  PGCD\;(28\;;\ 84)&=&1\times 2^{2}\times 7\\\\&=&28\end{array}$
 
Donc, $\boxed{PGCD\;(28\;;\ 84)=28}$
 
Comme $PGCD\;(28\;;\ 84)=28$ alors, on a :
 
$\dfrac{84}{28}=\dfrac{84\div 28}{28\div 28}=\dfrac{3}{1}$
 
D'où, $\boxed{\dfrac{84}{28}=3}$
 
Soit à simplifier $\dfrac{7}{21}$
 
On a : $\dfrac{7}{21}=\dfrac{7}{3\times 7}$
 
Simplifions alors par $7.$
 
Ce qui donne : $\boxed{\dfrac{7}{21}=\dfrac{1}{3}}$
 
Soit à simplifier $-\dfrac{120}{160}$
 
On a :
 
$\begin{array}{rcl} -\dfrac{120}{160}&=&-\dfrac{120\div 10}{160\div 10}\\\\&=&-\dfrac{12}{16}\\\\&=&-\dfrac{3\times 4}{4\times 4}\\\\&=&-\dfrac{3}{4}\end{array}$
 
Ainsi, $\boxed{-\dfrac{120}{160}=-\dfrac{3}{4}}$
 
Soit à simplifier $-\dfrac{-16}{-48}$
 
Comme $48=3\times 16$ alors, on a :
 
$\begin{array}{rcl} -\dfrac{-16}{-48}&=&-\dfrac{16}{48}\\\\&=&-\dfrac{16}{3\times 16}\\\\&=&-\dfrac{1}{3}\end{array}$
 
D'où, $\boxed{-\dfrac{-16}{-48}=-\dfrac{1}{3}}$
 
Soit à simplifier $\dfrac{210}{-441}$
 
On sait que : $441=21\times 21\ $ et $\ 210=21\times 10$
 
Donc,
 
$\begin{array}{rcl} \dfrac{210}{-441}&=&-\dfrac{210}{441}\\\\&=&-\dfrac{21\times 10}{21\times 21}\\\\&=&-\dfrac{10}{21}\end{array}$
 
2) Désignons ceux qui sont des opposés.
 
On rappelle que deux nombres rationnels $a\ $ et $\ b$ sont opposés si, et seulement si :
$$a+b=0$$
D'après le résultat de la question $1)$, on a :
 
$\dfrac{64}{192}=\dfrac{1}{3}\;;\quad \dfrac{7}{21}=\dfrac{1}{3}\ $ et $\ -\dfrac{-16}{-48}=-\dfrac{1}{3}$
 
Or, $\dfrac{1}{3}-\dfrac{1}{3}=0$
 
Donc, les nombres $\dfrac{64}{192}\ $ et $\ -\dfrac{-16}{-48}$ sont des opposés.
 
De même, les nombres rationnels $\dfrac{7}{21}\ $ et $\ -\dfrac{-16}{-48}$ sont des opposés.
 
3) Désignons sont ceux qui sont des inverses
 
On rappelle que deux nombres rationnels $a\ $ et $\ b$ sont inverses si, et seulement si :
$$a\times b=1$$
D'après le résultat de la question $1)$, on a :
 
$\dfrac{64}{192}=\dfrac{1}{3}\;;\quad \dfrac{7}{21}=\dfrac{1}{3}\ $ et $\ +\dfrac{84}{28}=3$
 
Or, $\dfrac{1}{3}\times 3=\dfrac{3}{3}=1$
 
Donc, les nombres $\dfrac{64}{192}\ $ et $\ +\dfrac{84}{28}$ sont des inverses.
 
De même, les nombres rationnels $\dfrac{7}{21}\ $ et $\ +\dfrac{84}{28}$ sont aussi des inverses.

Exercice 17

Calculons chacune des expressions suivantes en donnant le résultat sous forme de fractions irréductibles.
 
Soit : $A=\left(-\dfrac{8}{7}\right)+\left(-\dfrac{7}{14}\right)-\left(-\dfrac{3}{2}\right)$
 
Alors, en appliquant la règle de suppression des parenthèses et en réduisant au même dénominateur, on obtient :
 
$\begin{array}{rcl} A&=&\left(-\dfrac{8}{7}\right)+\left(-\dfrac{7}{14}\right)-\left(-\dfrac{3}{2}\right)\\\\&=&-\dfrac{8}{7}-\dfrac{7}{14}+\dfrac{3}{2}\\\\&=& -\dfrac{16}{14}-\dfrac{7}{14}+\dfrac{21}{14}\\\\&=&\dfrac{(-16-7+21)}{14}\\\\&=&\dfrac{-2}{14}\\\\&=&\dfrac{-2\div 2}{14\div 2}\\\\&=&\dfrac{-1}{7}\end{array}$
 
Ainsi, $\boxed{A=-\dfrac{1}{7}}$
 
Soit : $B=\dfrac{3}{7}-\dfrac{1}{7}\times\left(\dfrac{5}{2}-5\right)^{2}$
 
Alors, en calculant, on trouve :
 
$\begin{array}{rcl} B&=&\dfrac{3}{7}-\dfrac{1}{7}\times\left(\dfrac{5}{2}-5\right)^{2}\\\\&=&\dfrac{3}{7}-\dfrac{1}{7}\times\left(\dfrac{5}{2}-\dfrac{10}{2}\right)^{2}\\\\&=& \dfrac{3}{7}-\dfrac{1}{7}\times\left(\dfrac{5-10}{2}\right)^{2}\\\\&=&\dfrac{3}{7}-\dfrac{1}{7}\times\left(\dfrac{-5}{2}\right)^{2}\\\\&=&\dfrac{3}{7}-\left(\dfrac{1}{7}\times\dfrac{25}{4}\right)\\\\&=&\dfrac{3}{7}-\dfrac{25}{28}\\\\&=&\dfrac{12}{28}-\dfrac{25}{28}\\\\ &=&\dfrac{12-25}{28}\\\\&=&\dfrac{-13}{28}\end{array}$
 
D'où, $\boxed{B=-\dfrac{13}{28}}$
 
Soit : $C=\left|1-\dfrac{4}{3}\right|-\left|1+\dfrac{1}{2}\right|\times\left(-\dfrac{1}{2}\right)^{2}$
 
Alors, en appliquant les propriétés de la valeur absolue, on obtient :
 
$\begin{array}{rcl} C&=&\left|1-\dfrac{4}{3}\right|-\left|1+\dfrac{1}{2}\right|\times\left(-\dfrac{1}{2}\right)^{2}\\\\&=&\left|\dfrac{3}{3}-\dfrac{4}{3}\right|-\left|\dfrac{2}{2}+\dfrac{1}{2}\right|\times\dfrac{1}{4}\\\\&=&\left|\dfrac{3-4}{3}\right|-\left|\dfrac{2+1}{2}\right|\times\dfrac{1}{4}\\\\&=&\left|\dfrac{-1}{3}\right|-\left|\dfrac{3}{2}\right|\times\dfrac{1}{4}\\\\ &=&\dfrac{1}{3}-\dfrac{3}{2}\times\dfrac{1}{4}\\\\&=&\dfrac{1}{3}-\dfrac{3}{8}\\\\&=&\dfrac{8}{24}-\dfrac{9}{24}\\\\&=&\dfrac{8-9}{24}\\\\&=&\dfrac{-1}{24}\end{array}$
 
Donc, $\boxed{C=-\dfrac{1}{24}}$
 
Soit : $D=\left(\dfrac{4-(2-5)^{2}}{7-5}\right)^{3}+\dfrac{17}{8}$
 
Alors, en calculant, on obtient :
 
$\begin{array}{rcl} D&=&\left(\dfrac{4-(2-5)^{2}}{7-5}\right)^{3}+\dfrac{17}{8}\\\\&=&\left(\dfrac{4-(-3)^{2}}{2}\right)^{3}+\dfrac{17}{8}\\\\&=&\left(\dfrac{4-9}{2}\right)^{3}+\dfrac{17}{8}\\\\&=&\left(\dfrac{-5}{2}\right)^{3}+\dfrac{17}{8}\\\\&=&\dfrac{(-5)^{3}}{2^{3}}+\dfrac{17}{8}\\\\&=&\dfrac{-125}{8}+\dfrac{17}{8}\\\\&=&\dfrac{-125+17}{8}\\\\&=&\dfrac{-108}{8}\\\\&=&\dfrac{-108\div 4}{8\div 4}\\\\&=&\dfrac{-27}{2}\end{array}$
 
D'où, $\boxed{D=-\dfrac{27}{2}}$

Exercice 18

Sachant que : $a=-\dfrac{5}{2}\;;\ b=\dfrac{3}{2}\;;\ c=\dfrac{1}{2}\ $ et $\ d=\dfrac{1}{6}$ calculons puis rendons irréductible le résultat de :
$$X=\dfrac{a+b}{b-d}\;;\quad Y=a\times c+b\div d\ \text{ et }\ Z=(b-a+c)^{2}$$
Soit : $X=\dfrac{a+b}{b-d}$
 
En remplaçant $a\;;\ b\ $ et $\ d$ par leur valeur, on obtient :
 
$\begin{array}{rcl} X&=&\dfrac{a+b}{b-a}\\\\&=&\dfrac{-\dfrac{5}{2}+\dfrac{3}{2}}{\dfrac{3}{2}-\dfrac{1}{6}}\\\\&=& \dfrac{-\dfrac{2}{2}}{\dfrac{9}{6}-\dfrac{1}{6}}\\\\&=&\dfrac{-1}{\dfrac{8}{6}}\\\\&=&\dfrac{-6}{8}\\\\&=&\dfrac{-6\div 2}{8\div 2}\\\\&=&\dfrac{-3}{4}\end{array}$
 
Ainsi, $\boxed{X=-\dfrac{3}{4}}$
 
Soit : $Y=a\times c+b\div d$
 
Remplaçons $a\;;\ b\;;\ c $ et $\ d$ par leur valeur.
 
On obtient alors :
 
$\begin{array}{rcl} Y&=&a\times c+b\div d\\\\&=&\left(-\dfrac{5}{2}\times\dfrac{1}{2}\right)+\left(\dfrac{3}{2}\div\dfrac{1}{6}\right)\\\\&=&-\dfrac{5}{4}+\left(\dfrac{3}{2}\times\dfrac{6}{1}\right)\\\\&=& \dfrac{-5}{4}+\dfrac{18}{2}\\\\&=&-\dfrac{5}{4}+\dfrac{36}{4}\\\\&=&\dfrac{-5+36}{4}\\\\&=&\dfrac{31}{4}\end{array}$
 
Donc, $\boxed{Y=\dfrac{31}{4}}$
 
Soit : $Z=(b-a+c)^{2}$
 
En remplaçant $a\;;\ b\ $ et $\ c$ par leur valeur, on obtient :
 
$\begin{array}{rcl} Z&=&(b-a+c)^{2}\nonumber\\ &=&\left(\dfrac{3}{2}-\left(-\dfrac{5}{2}\right)+\dfrac{1}{2}\right)^{2}\\\\&=&\left(\dfrac{3}{2}+\dfrac{5}{2}+\dfrac{1}{2}\right)^{2}\\\\&=&\left(\dfrac{3+5+1}{2}\right)^{2}\\\\&=&\left(\dfrac{9}{2}\right)^{2}\\\\&=&\dfrac{9^{2}}{2^{2}}\\\\&=&\dfrac{81}{4}\end{array}$
 
D'où, $\boxed{Z=\dfrac{81}{4}}$

Exercice 19

Calculons chacune des expressions suivantes en donnant le résultat sous forme de fractions irréductibles.
 
Soit : $A=\dfrac{1+\dfrac{1}{3}}{1-\dfrac{1}{3}}$
Alors, en calculant, on obtient :
 
$\begin{array}{rcl} A&=&\dfrac{1+\dfrac{1}{3}}{1-\dfrac{1}{3}}\\\\&=&\dfrac{\dfrac{3}{3}+\dfrac{1}{3}}{\dfrac{3}{3}-\dfrac{1}{3}}\\\\&=&\dfrac{\dfrac{3+1}{3}}{\dfrac{3-1}{3}}\\\\&=&\dfrac{\dfrac{4}{3}}{\dfrac{2}{3}}\\\\&=&\dfrac{4}{3}\times\dfrac{3}{2}\\\\&=&\dfrac{4}{2}\\\\&=&2\end{array}$
 
Donc, $\boxed{A=2}$
 
Soit : $B=\dfrac{2^{2}+\dfrac{3}{4}}{-5+\dfrac{3}{4}}$
 
En calculant, on trouve :
 
$\begin{array}{rcl} B&=&\dfrac{2^{2}+\dfrac{3}{4}}{-5+\dfrac{3}{4}}\\\\&=& \dfrac{4+\dfrac{3}{4}}{-5+\dfrac{3}{4}}\\\\&=&\dfrac{\dfrac{16}{4}+\dfrac{3}{4}}{\dfrac{-20}{4}+\dfrac{3}{4}}\\\\&=&\dfrac{\dfrac{16+3}{4}}{\dfrac{-20+3}{4}}\\\\&=&\dfrac{\dfrac{19}{4}}{\dfrac{-17}{4}}\\\\&=&\dfrac{19}{4}\times\dfrac{4}{-17}\\\\&=&\dfrac{19}{-17}\end{array}$
 
D'où, $\boxed{B=-\dfrac{19}{17}}$
 
Soit : $C=\dfrac{1-\dfrac{1}{3}}{2+\dfrac{1}{4}}\div\dfrac{2-\dfrac{1}{4}}{1+\dfrac{1}{3}}$
 
Alors, en calculant, on obtient :
 
$\begin{array}{rcl} C&=&\dfrac{1-\dfrac{1}{3}}{2+\dfrac{1}{4}}\div\dfrac{2-\dfrac{1}{4}}{1+\dfrac{1}{3}}\\\\&=&\dfrac{\dfrac{3}{3}-\dfrac{1}{3}}{\dfrac{8}{4}+\dfrac{1}{4}}\div\dfrac{\dfrac{8}{4}-\dfrac{1}{4}}{\dfrac{3}{3}+\dfrac{1}{3}}\\\\&=&\dfrac{\dfrac{3-1}{3}}{\dfrac{8+1}{4}}\div\dfrac{\dfrac{8-1}{4}}{\dfrac{3+1}{3}}\\\\ &=& \dfrac{\dfrac{2}{3}}{\dfrac{9}{4}}\div\dfrac{\dfrac{7}{4}}{\dfrac{4}{3}}\\\\ &=& \left(\dfrac{2}{3}\times\dfrac{4}{9}\right)\div\left(\dfrac{7}{4}\times\dfrac{3}{4}\right)\\\\ &=& \left(\dfrac{8}{27}\right)\div\left(\dfrac{21}{16}\right)\\\\ &=& \dfrac{8}{27}\times\dfrac{16}{21}\\\\ &=& \dfrac{128}{567}\end{array}$
 
Ainsi, $\boxed{C=\dfrac{128}{567}}$
 
Soit : $F=\dfrac{1+\dfrac{2\pi}{3}}{4-\dfrac{3}{2\pi}}$
 
En calculant, on obtient :
 
$\begin{array}{rcl} F&=& \dfrac{1+\dfrac{2\pi}{3}}{4-\dfrac{3}{2\pi}}\\\\&=&\dfrac{\dfrac{3}{3}+\dfrac{2\pi}{3}}{\dfrac{8\pi}{2\pi}-\dfrac{3}{2\pi}}\\\\&=&\dfrac{\dfrac{3+2\pi}{3}}{\dfrac{8\pi-3}{2\pi}}\\\\&=& \dfrac{3+2\pi}{3}\times\dfrac{2\pi}{8\pi-3}\\\\&=&\dfrac{(3+2\pi)(2\pi)}{3(8\pi-3)}\\\\&=&\dfrac{6\pi+4\pi^{2}}{24\pi-9}\end{array}$
 
Donc, $\boxed{F=\dfrac{6\pi+4\pi^{2}}{24\pi-9}}$

Exercice 20

Calculons puis rendons irréductible.
 
Soit : $A=\dfrac{\dfrac{1}{3}}{\dfrac{2}{7}\times\dfrac{1}{4}}+\dfrac{\dfrac{\dfrac{1}{2}}{5}\times\dfrac{1}{8}}{\dfrac{1}{7}\div\dfrac{3}{\dfrac{5}{2}\times 4}}$
 
Alors, en calculant, on trouve :
 
$\begin{array}{rcl} A&=&\dfrac{\dfrac{1}{3}}{\dfrac{2}{7}\times\dfrac{1}{4}}+\dfrac{\dfrac{\dfrac{1}{2}}{5}\times\dfrac{1}{8}}{\dfrac{1}{7}\div\dfrac{3}{\dfrac{5}{2}\times 4}}\\\\&=&\dfrac{\dfrac{1}{3}}{\dfrac{2}{28}}+\dfrac{\dfrac{1}{2}\times\dfrac{1}{5}\times\dfrac{1}{8}}{\dfrac{1}{7}\div\dfrac{3}{\dfrac{20}{2}}}\\\\&=&\dfrac{1}{3}\times\dfrac{28}{2}+\dfrac{\dfrac{1}{80}}{\dfrac{1}{7}\div\dfrac{3}{10}}\\\\&=&\dfrac{28}{6}+\dfrac{\dfrac{1}{80}}{\dfrac{1}{7}\times\dfrac{10}{3}}\\\\&=&\dfrac{14}{3}+\dfrac{\dfrac{1}{80}}{\dfrac{10}{21}}\\\\&=&\dfrac{14}{3}+\left(\dfrac{1}{80}\times\dfrac{21}{10}\right)\\\\&=&\dfrac{14}{3}+\dfrac{21}{800}\\\\&=&\dfrac{11\,200}{24\,000}+\dfrac{63}{24\,000}\\\\&=&\dfrac{11\,263}{24\,000}\end{array}$
 
D'où, $\boxed{A=\dfrac{11\,263}{24\,000}}$
 
Soit : $B=\dfrac{(-2)^{2}\times\dfrac{5}{3}}{7-\dfrac{2}{3}}\div\dfrac{(-1)^{9}+\dfrac{4}{9}}{1-\dfrac{2}{11}}$
 
On rappelle que si $n$ est un entier naturel alors :
 
$\ \centerdot\ (-1)^{n}=1$ si $n$ est pair ; c'est-à-dire, si $n$ est un multiple de $2$
 
$\ \centerdot\ (-1)^{n}=-1$ si $n$ est impair ; c'est-à-dire, si $n$ n'est pas multiple de $2$
 
Alors, en calculant, on trouve :
 
$\begin{array}{rcl} B&=&\dfrac{(-2)^{2}\times\dfrac{5}{3}}{7-\dfrac{2}{3}}\div\dfrac{(-1)^{9}+\dfrac{4}{9}}{1-\dfrac{2}{11}}\\\\&=&\dfrac{4\times\dfrac{5}{3}}{\dfrac{21}{3}-\dfrac{2}{3}}\div\dfrac{-1+\dfrac{4}{9}}{\dfrac{11}{11}-\dfrac{2}{11}}\\\\&=&\dfrac{\dfrac{20}{3}}{\dfrac{21-2}{3}}\div\dfrac{\dfrac{-9}{9}+\dfrac{4}{9}}{\dfrac{11-2}{11}}\\\\&=&\dfrac{\dfrac{20}{3}}{\dfrac{19}{3}}\div\dfrac{\dfrac{-9+4}{9}}{\dfrac{9}{11}}\\\\&=&\left(\dfrac{20}{3}\times\dfrac{3}{19}\right)\div\dfrac{\dfrac{-5}{9}}{\dfrac{9}{11}}\\\\&=&\dfrac{20}{19}\div\left(\dfrac{-5}{9}\times\dfrac{11}{9}\right)\\\\&=&\dfrac{20}{19}\div\left(\dfrac{-55}{81}\right)\\\\&=&\dfrac{20}{19}\times\dfrac{81}{-55}\\\\&=&\dfrac{4}{19}\times\dfrac{81}{-11}\\\\&=&\dfrac{324}{-209}\end{array}$
 
Donc, $\boxed{B=-\dfrac{324}{209}}$
 
Soit : $C=\dfrac{\dfrac{1}{3}}{\dfrac{2}{7}\times\dfrac{1}{4}}-\dfrac{\dfrac{4}{5}\times\dfrac{1}{8}}{\dfrac{1}{7}\times\dfrac{3}{4}}$
 
Alors, en calculant, on trouve :
 
$\begin{array}{rcl} C&=&\dfrac{\dfrac{1}{3}}{\dfrac{2}{7}\times\dfrac{1}{4}}-\dfrac{\dfrac{4}{5}\times\dfrac{1}{8}}{\dfrac{1}{7}\times\dfrac{3}{4}}\\\\&=&\dfrac{\dfrac{1}{3}}{\dfrac{2}{28}}-\dfrac{\dfrac{4}{40}}{\dfrac{3}{28}}\\\\&=&\left(\dfrac{1}{3}\times\dfrac{28}{2}\right)-\left(\dfrac{4}{40}\times\dfrac{28}{3}\right)\\\\&=&\left(\dfrac{1}{3}\times\dfrac{14}{1}\right)-\left(\dfrac{1}{10}\times\dfrac{28}{3}\right)\\\\&=&\dfrac{14}{3}-\dfrac{14}{15}\\\\&=&\dfrac{70}{15}-\dfrac{14}{15}\\\\&=&\dfrac{56}{15}\end{array}$
 
Ainsi, $\boxed{C=\dfrac{56}{15}}$
 
Soit : $D=\dfrac{\dfrac{1}{3}}{\dfrac{2}{7}+\dfrac{1}{4}}\times\dfrac{\dfrac{\dfrac{1}{4}}{5}-\dfrac{1}{8}}{\dfrac{1}{7}-\dfrac{3}{\dfrac{5}{2}+4}}$
 
Alors, en calculant, on obtient :
 
$\begin{array}{rcl} D&=&\dfrac{\dfrac{1}{3}}{\dfrac{2}{7}+\dfrac{1}{4}}\times\dfrac{\dfrac{\dfrac{1}{4}}{5}-\dfrac{1}{8}}{\dfrac{1}{7}-\dfrac{3}{\dfrac{5}{2}+4}}\\\\&=&\dfrac{\dfrac{1}{3}}{\dfrac{8}{28}+\dfrac{7}{28}}\times\dfrac{\dfrac{1}{4}\times\dfrac{1}{5}-\dfrac{1}{8}}{\dfrac{1}{7}-\dfrac{3}{\dfrac{5}{2}+\dfrac{8}{2}}}\\\\&=&\dfrac{\dfrac{1}{3}}{\dfrac{15}{28}}\times\dfrac{\dfrac{1}{20}-\dfrac{1}{8}}{\dfrac{1}{7}-\dfrac{3}{\dfrac{13}{2}}}\\\\&=&\dfrac{1}{3}\times\dfrac{28}{15}\times\dfrac{\dfrac{8}{160}-\dfrac{20}{160}}{\dfrac{1}{7}-\dfrac{3}{1}\times\dfrac{2}{13}}\\\\&=&\dfrac{28}{45}\times\dfrac{\dfrac{-12}{160}}{\dfrac{1}{7}-\dfrac{6}{13}}\\\\&=&\dfrac{28}{45}\times\dfrac{\dfrac{-12}{160}}{\dfrac{13}{91}-\dfrac{42}{91}}\\\\&=&\dfrac{28}{45}\times\dfrac{\dfrac{-12}{160}}{\dfrac{-29}{91}}\\\\&=&\dfrac{28}{45}\times\left(\dfrac{-12}{160}\times\dfrac{91}{-29}\right)\\\\&=&\dfrac{28}{45}\times\dfrac{12}{160}\times\dfrac{91}{29}\\\\&=&\dfrac{4\times 7\times 3\times 4\times 91}{3\times 15\times 4\times 4\times 10\times 29}\\\\&=&\dfrac{7\times 91}{15\times 10\times 29}\\\\&=&\dfrac{637}{4\,350}\end{array}$
 
D'où, $\boxed{D=\dfrac{637}{4\,350}}$

Exercice 21

On considère les encadrements suivants :
$$1.720<x<1.721\ \text{ et }\ 1.5<y<1.51$$
a) Donnons un encadrement d'ordre $1$ de $x+y.$
 
Cela signifie d'encadrer $x+y$ par deux nombres décimaux à un chiffre après la virgule.
 
On a :
$$\begin{array}{rcccl} 1.720&<&x&<&1.721\\1.5&<&y&<&1.51 \end{array}$$
Alors, en additionnant membre à membre ces deux encadrements, on obtient :
$$\begin{array}{rcccl} 1.720&<&x&<&1.721\\\\+\quad 1.5&<&y&<&1.51\\\\\hline \\=\quad 3.22&<&x+y&<&3.231\end{array}$$
Ainsi, un encadrement d'ordre $1$ de $x+y$ est donné par :
$$\boxed{3.2<x+y<3.3}$$
b) Donnons un encadrement d'ordre $2$ de $x-y$ puis en déduisons sa valeur approchée par défaut.
 
Cela revient à encadrer $x-y$ par deux nombres décimaux à deux chiffres après la virgule.
 
On va d'abord encadre $(-y)$ en multipliant chaque membre de l'encadrement de $y$ par le même nombre $-1.$
 
On rappelle que les inégalités changent de sens lorsqu'on multiplie l'encadrement par un nombre négatif.
 
Comme $1.5<y<1.51$ alors, on a : $-1.51<-y<-1.5$
 
Ainsi, on obtient : 
$$\begin{array}{rcccl} 1.720&<&x&<&1.721\\-1.51&<&-y&<&-1.5 \end{array}$$
Alors, en additionnant membre à membre ces deux encadrements, on obtient :
$$\begin{array}{rcccl} 1.720&<&x&<&1.721\\\\-1.51&<&-y&<&-1.5\\\\\hline \\=\quad 0.21&<&x+y&<&0.221\end{array}$$
D'où, un encadrement d'ordre $2$ de $x-y$ est donné par :
$$\boxed{0.21<x-y<0.22}$$
Par conséquent, la valeur approchée par défaut de $x-y$ est égale à : $0.21$

Exercice 22

On considère les encadrements suivants :
$$3.80<x<3.81\ \text{ et }\ 1.5<y<1.51$$
1) Donnons un encadrement de $3x+2y$ à $10^{-1}$ prés puis en déduisons sa valeur approchée par excès.
 
On commence par encadrer $3x$ en multipliant chaque membre de l'encadrement de $x$ par le même nombre $3.$
 
On obtient alors : $3\times 3.80<3\times x<3\times 3.81$
 
Ce qui donne : $11.4<3x<11.43$
 
Ensuite, on encadre $2y$ en multipliant chaque membre de l'encadrement de $y$ par le même nombre $2.$
 
On obtient alors : $2\times 1.5<2y<2\times 1.51$
 
Ce qui donne : $3<2y<3.02$
 
Ainsi, on a :
$$\begin{array}{rcccl} 11.4&<&3x&<&11.43\\3&<&2y&<&3.02\end{array}$$
Par suite, en additionnant membre à membre ces deux encadrements, on obtient :
$$\begin{array}{rcccl} 11.4&<&3x&<&11.43\\\\+\quad 3&<&2y&<&3.02\\\\\hline \\=\quad 14.4&<&3x+2y&<&14.45\end{array}$$
Ainsi, un encadrement de $3x+2y$ à $10^{-1}$ près est donné par :
$$\boxed{14.4<3x+2y<14.5}$$
Sa valeur approchée par excès est donc égale à $14.5$
 
2) Donnons un encadrement de $2x-3y$ à $10^{-2}$ prés.
 
On commence par encadrer $(-3y)$ en multipliant chaque membre de l'encadrement de $y$ par le même nombre $-3.$
 
On rappelle que les inégalités changent de sens lorsqu'on multiplie l'encadrement par un nombre négatif.
 
Comme $1.5<y<1.51$ alors, on a : $-3\times 1.51<-3\times y<-3\times 1.5$
 
Ce qui donne : $-4.53<-3y<-4.5$
 
Ensuite, on encadre $2x$ en multipliant chaque membre de l'encadrement de $x$ par le même nombre $2.$
 
On obtient alors : $2\times 3.80<2x<2\times 3.81$
 
Ce qui donne : $7.6<2x<7.62$
 
Ainsi, on a : 
$$\begin{array}{rcccl} 7.6&<&2x&<&7.62\\-4.53&<&-3y&<&-4.5\end{array}$$
Alors, en additionnant membre à membre ces deux encadrements, on obtient :
$$\begin{array}{rcccl} 7.6&<&2x&<&7.62\\\\-4.53&<&-3y&<&-4.5\\\\\hline \\=\quad 3.07&<&2x-3y&<&3.12\end{array}$$
D'où, un encadrement de $2x-3y$ à $10^{-2}$ prés est donné par :
$$\boxed{3.07<2x-3y<3.12}$$
3) Donnons un encadrement de $\dfrac{x}{y}$ à $10^{-1}$ prés.
 
On commence par encadrer $\dfrac{1}{y}$ en remplaçant chaque membre de l'encadrement de $y$ par son inverse tout en changeant le sens des inégalités.
 
Comme $1.5<y<1.51$ alors, on a : $\dfrac{1}{1.51}<\dfrac{1}{y}<\dfrac{1}{1.5}$
 
Ensuite, on multiplie membre à membre l'encadrement de $x$ par celui de $\dfrac{1}{y}.$
 
On obtient : $3.80\times\dfrac{1}{1.51}<x\times\dfrac{1}{y}<3.81\times\dfrac{1}{1.5}$
 
Ce qui donne : $\dfrac{3.80}{1.51}<\dfrac{x}{y}<\dfrac{3.81}{1.5}$
 
On trouve alors : $2.51<\dfrac{x}{y}<2.54$
 
Ainsi, un encadrement de $\dfrac{x}{y}$ à $10^{-1}$ prés est donné par :
$$\boxed{2.5<\dfrac{x}{y}<2.6}$$

Exercice 23

On considère un rectangle dont les dimensions en $cm$ sont $3\ $ et $\ x-4.$
 
On suppose que : $10\leq x<15.$
 
Donnons un encadrement de l'aire $A$ en $cm^{2}$ de ce rectangle d'amplitude la plus petite possible.
 
Comme les dimensions sont $3\ $ et $\ x-4$ alors, l'aire $A$ de ce rectangle est donnée par :
$$A=3\times(x-4)$$
Or, $10\leq x<15.$
 
Donc, $10-4\leq x-4<15-4.$
 
Ce qui donne : $6\leq x-4<11.$
 
En multipliant chaque membre de cet encadrement par le même nombre $3$, on obtient : $3\times 6\leq 3\times(x-4)<3\times 11.$
 
Donc, $18\leq 3\times(x-4)<33.$
 
Ainsi, un encadrement de l'aire $A$ en $cm^{2}$ de ce rectangle est donné par :
$$\boxed{18\;cm^{2}\leq A<33\;cm^{2}}$$

Exercice 24

Soient $x\ $ et $\ y$ deux nombres rationnels tels que :
$$x=\dfrac{7934}{934}\ \text{ et }\ y=\dfrac{3794}{973}$$
1) Trouvons les entiers $a\ $ et $\ b$ tels que :
$$a\leq x<a+1\ \text{ et }\ b\leq y<b+1$$
Par calcul direct, on trouve : $x=\dfrac{7934}{934}=8.49$
 
Or, $8<8.49<9$
 
Donc, $8<x<8+1$
 
Par conséquent, $\boxed{a=8}$
 
De la même manière, on a : $y=\dfrac{3794}{973}\simeq 3.899$
 
Or, $3<3.899<4$
 
Ce qui signifie que : $3<y<3+1$
 
Ainsi, $\boxed{b=3}$
 
2) Donnons un encadrement de $x+y$
 
D'après le résultat de la question $1)$, on a :
$$\begin{array}{rcccl} 8&<&x&<&9\\3&<&y&<&4\end{array}$$
Alors, en additionnant membre à membre ces deux encadrements, on obtient :
$$\begin{array}{rcccl} 8&<&x&<&9\\\\+\quad 3&<&y&<&4\\\\\hline \\=\quad 11&<&x+y&<&13\end{array}$$
Ainsi, un encadrement de $x+y$ est donné par :
$$\boxed{11<x+y<13}$$
 
 

Auteur: 
Diny Faye

Commentaires

j aime les matières scientifiques

Très bien

AU TEMP QUE MOI

Valeurs absolue

Merci je révise en même temps

En expliquant

Bonjour je voulais avoir la correction de la série pour corriger mes éventuelles erreurs. Merci Messieurs

Merci pour la correction mais ke pense que dans l'exercice3 à la question b) à l'opération D il y'a une erreur de calcul on doit trouver normalement -10/7 MERCi pour votre soutien

Les signes

Très bien pour exercice 7 correction A

Très bien pour exercice 7 correction A

Qui peut m'aider pour l'exercice 8 9 et 10

Ajouter un commentaire